JustinD731
Thanks Received: 0
Vinny Gambini
Vinny Gambini
 
Posts: 1
Joined: December 03rd, 2017
 
 
 

Addressing the "OR" in Relative Ordering

by JustinD731 Fri Nov 16, 2018 7:05 pm

Hello All,

I just went over my answers for LG Set number 2, page 53 1-6 and I got most of my answers wrong. I am a little confused when presented with rules in a problem where you see "or". I know that usually means drawing two different trees, which isn't a problem. But, my question is, when a problem asks if an element can or cannot go into a specific spot on a number line (i.e. Mertz's design cannot be presented first, second, third, etc.) does the answer have to be true for both trees drawn or could the answer be correct if it fulfills the requirements one of the trees drawn?
User avatar
 
ohthatpatrick
Thanks Received: 3805
Atticus Finch
Atticus Finch
 
Posts: 4661
Joined: April 01st, 2011
 
 
 

Re: Addressing the "OR" in Relative Ordering

by ohthatpatrick Sat Nov 17, 2018 1:32 am

I'm a little confused by what you're specifically asking, so feel free to clarify.

"X or Y" on LSAT should be interpreted as "at least one of these", unless we are specifically told "but not both".

As it relates to PT53,
rule 2 tells us that in every legal scenario of the game, G will either be BEFORE J and L or AFTER J and L.
rule 3 says in every legal scenario of the game, V will be BEFORE G and P or AFTER G and P.

When you're doing a question like Q7, it's just saying "which spot does M never show up in, in this game".

One of these spots is a spot Mertz CANNOT ever be in.
The other four of these are spots that Mertz CAN be in.

M can NEVER be in 6, because L has to still come after M. That's why (A) is correct.

The others are all possible placements for M (and many of us do all the "If" questions before we do unconditional questions like Q7, so that we can simply look back at all our previous work and take note of what positions we've seen M in, in our various hypotheticals).

For M to be in 5, we know that L would be in 6.
Rule 2 told us that G is either BEFORE J and L or AFTER J and L.
Well, if L is in 6, then G is not going to be AFTER J and L.
Hence, if M is to be in 5, then G will have to be BEFORE J and L.

M couldn't be 5 in both of your frames for rule 2, but those two frames contradict each other. You're not going to have G simultaneously be before AND after J and L. Anything compatible with BEFORE J and L could understandably be incompatible with AFTER J and L.

For a scenario to be possible (a could be true), it just has to not-break any rules.

If you write a scenario like
V G P J M L
does it break any rules?

rule 1 is fine - M is before L but after P
rule 2 is fine - G is either before both of J/L or after both of them (in this case before)
rule 3 is fine - V is either before both of G/P or after both of them (in this case before)

Since the V G P J M L scenario doesn't break any rules, it could be true.
Thus, M could be fifth.

Does that make sense?